LSAT and Law School Admissions Forum

Get expert LSAT preparation and law school admissions advice from PowerScore Test Preparation.

User avatar
 Dave Killoran
PowerScore Staff
  • PowerScore Staff
  • Posts: 5852
  • Joined: Mar 25, 2011
|
#88063
Complete Question Explanation
(The complete setup for this game can be found here: lsat/viewtopic.php?f=165&t=7597)

The correct answer choice is (D).

The question stem specifies that there are six messages sent, and that L sent the fifth message. Thus, only Template 3A applies:

G2-Q11-d1.png

In this template, however, when L sends the fifth message, then J must send the fourth message:

G2-Q11-d2.png

Accordingly, answer choice (D) is correct.
You do not have the required permissions to view the files attached to this post.
 cls
  • Posts: 14
  • Joined: Oct 02, 2013
|
#13116
Could you go through number 11 please? Both answer choices C and D are seeming possible to me.

Thanks.
 David Boyle
PowerScore Staff
  • PowerScore Staff
  • Posts: 836
  • Joined: Jun 07, 2013
|
#13127
cls wrote:Could you go through number 11 please? Both answer choices C and D are seeming possible to me.

Thanks.
Dear cls:

For answer choice C, it seems that hjljlh would work. Obviously, this would show that message 3 need not be from h.
On a broader level: since l is 5th, it seems that someone else must be 6th, such as h. I just plugged h into 6th place, and 1st place, and then followed the rules after that.
Good luck on the test!

Hope this helps,
David
 ccampise
  • Posts: 27
  • Joined: Jul 14, 2014
|
#15958
For question 11 I'm trying to figure out why J must be fourth? It seems the hypothetical HJLLJH would prove that it doesn't have to be? I'm sure I'm missing something here. If anybody has the answer feel free to chime in!
 Nikki Siclunov
PowerScore Staff
  • PowerScore Staff
  • Posts: 1362
  • Joined: Aug 02, 2011
|
#15980
Hi,

The condition in this Local question is that we have 6 messages, the fifth one of which is left by L:

_ _ _ _ L _

(double-check your local set-up - you did not satisfy the local condition!)

From here on, the are probably two ways of solving the question, depending on whether it is J or H that leave the first (and the last) messages:

J _ _ _ L J

or

H _ _ _ L H

Clearly, the first solution wouldn't work, because it cannot accommodate an HJ block. Thus, we are left with the second solution:

H J _ _ L H

Due to the last rule requiring us to have exactly one J in the first three spaces, the third message must be left by L. And, since each person leaves exactly two messages in a 2-2-2 distribution (for a total of six messages), the fourth message must be left by J:

H J L J L H

Let me know if this makes sense!

Thanks!
 ccampise
  • Posts: 27
  • Joined: Jul 14, 2014
|
#15982
Oh wow, thought I had L in the 5th spot. Guess I was staring at it too long! Thank you!

Get the most out of your LSAT Prep Plus subscription.

Analyze and track your performance with our Testing and Analytics Package.